subject
Physics, 30.11.2019 04:31 brifrog13

My notes a flat loop of wire consisting of a single turn of cross-sectional area 7.80 cm2 is perpendicular to a magnetic field that increases uniformly in magnitude from 0.500 t to 3.20 t in 1.02 s. what is the resulting induced current if the loop has a resistance of 1.30 ?

ansver
Answers: 1

Another question on Physics

question
Physics, 22.06.2019 15:10
What does si stand for,when referring to a system of measurement
Answers: 1
question
Physics, 22.06.2019 19:30
The nuclear potential that binds protons and neutrons in the nucleus of an atom is often approximated by a square well. imagine a proton conned in an innite square well of length 105 nm, a typical nuclear diameter. calculate the wavelength and energy associated with the photon that is emitted when the proton undergoes a transition from the rst excited state (n 2) to the ground state (n 1). in what region of the electromagnetic spectrum does this wavelength belong?
Answers: 1
question
Physics, 22.06.2019 21:50
Aforce of 8,480 n is applied to a cart to accelerate it at a rate of 26.5 m/s2. what is the mass of the cart?
Answers: 1
question
Physics, 22.06.2019 23:50
Which property is expressed in distance units? o a. period o b. wavelength o c. speed o d. frequency
Answers: 2
You know the right answer?
My notes a flat loop of wire consisting of a single turn of cross-sectional area 7.80 cm2 is perpend...
Questions
question
History, 21.04.2021 19:50
question
Mathematics, 21.04.2021 19:50
question
Health, 21.04.2021 19:50
question
Mathematics, 21.04.2021 19:50
Questions on the website: 13722363